Questions NB1

Réussis tes devoirs et examens dès maintenant avec Quizwiz!

A presynaptic GABAergic neuron (I.e., neuron that releases GABA) communicates with a post synaptic neuron that express3s dendritic GABAa receptors. The GABAa receptor is directly linked to a Cl- conducting transmembrane channel. If the postsynaptic neuron has a RMP of Vm = -60 mV and E(Cl-) is -65 mV, would the presynaptic neuron exert excitatory or inhibitory effects of postsynaptic neuron?

-60 is your RMP, IF YOU HAVE AN ION THATS MORE POSITIVE, you have more of an AP and less negative/more positive = depolarization = excitatory However, in this case Cl- has a conductance of -65 mV so it's MORE NEGATIVE/LESS POSITIVE, so it's gonna hyper polarize and not fire = inhibitory

How is MG different from LE syndrome?

-MG, more waning on EMG, but waxing for LE -MG symptoms better with rest, LE better with movement -MG worse during the day, LE better during the day -MG better during the night, LE worse during the night -MG antibodies with NAChR and LE antibodies with voltage gated Ca2+ channels -MG postsynaptic receptors, LE presynaptic receptors -LE related to oat cell carcinoma; ME related to thymus gland tumor -Both chronic autoimmune diseases and at the NMJ (MG at end plate and LE at nerve terminal) -MG Tx: remove thymus, ME Tx: remove tumor and give 4-aminopyrimidine

What is the main site of CSF production? What is the main site of CSF removal? What does this imply for the direction of CSF flow? Write out the CSF flow.

-Made in coracoid plexus; lines the ventricles and found at floor of lateral ventricle, the roof of the 3rd ventricle -Secreted in arachnoid villi -Implied that flow is unidirectional from production to drainage site Flow: lateral ventricle —> anterior horn —> foramen of Monroe —> 3rd ventricle —> cerebral aqueduct of sylvius—> 4th ventricle —> can exit in subarachnoid space or continues inferior to central canal, but if not it passes to foramen of Magendie and if left/right passing it goes to Foramen of Luschka to subarachnoid space —> arachnoid villi

Aminoglycoside antibiotics block voltage-gated Ca2+ channels. Which of the following effects on transmitter release from lower motor neurons would be seen in the presence of the drugs? A. Increased quantal content B. Decreased quantal content C. Increased quantal size D. Decreased quantal size

B. Decreased quantal content Less Ca2+ coming in so less vesicles being broken down to release NT, so you're decreasing quintal content

Explain the 3 functions of the muscle spindle

1. Participates in the stretch (myotatic) reflexes 2. Provides proprioceptive info to the CNS 3. Regulates motor contraction under the influence of descending motor pathways and afferent inputs

A 4-year-old boy swallows a piece of rodent poison that contains strychnine. Which of the following effects is most likely? A. Depression B. Blindness C. Muscular rigor D. Loss of smell E. Flaccid paralysis

C. Muscular rigor - more excitatory influence = more muscle contraction

A 28 year-old woman comes to the clinic complaining of dizzy spells for 3 months. Physical exam shows impaired balance. The patient's condition is found to be caused by free-floating calcium crystals found within the inner ear. The crystalline bodies most likely originated from which of the following structures? A. Crista Ampullaris B. Macule of the utricle C. Cochlear Duct D. Modiolus E. Oval Window

B. Macule of the utricle

Identify the major subarachnoid vessel normally supplying blood to the lenticulostriate arteries. 1. PICA 2. AICA 3. SCA 4. PCA 5. MCA 6. ACA

5. MCA

NO is released from one neuron to signal to another at a synapse. Which of the following are features of this signaling molecule? A. NO is formed form citrulline B. NO increased cGMP concentration in target cells C. NO activated PKA D. NO is formed from heme E. NO is formed from arginine

B. NO increased cGMP concentration in target cells

Choose the correct statements A. 5-HIAA is formed from serotonin by action of MAO B. Degradation of dopamine is by PNMT C. Tyrosine hydroxylase requires BH4 D. The precursor of dopamine is tryptophan E. The precursor of serotonin is tryptophan

A. 5-HIAA is formed from serotonin by action of MAO C. Tyrosine hydroxylase requires BH4 (also in TRP hydroxylase for synthesis of serotonin and PHE hydroxylase) E. The precursor of serotonin is tryptophan -B helps convert norepinephrine to epinephrine with SAM and is activated by cortisol

A newborn boy (32 weeks of gestation) is examined because of a postnatal seizure. A germline mutation of the homeobox gene EMX2 is suspected. Which of the following neuroimages best corresponds to this patient? A. A B. B C. C D. D

A. A

A 36 year old man is brought to the physician because of weakness of the lower limbs for 3 weeks. Neurological examination show symptoms of a lower motor neuron injury. Which of the following structures in the spinal cord is likely affected? A. Anterior horn B. Posterior horn C. Dorsal column D. Anterior white commissure

A. Anterior horn

The corticospinal and corticobulbar tracts originate in which of the following structures? A. Cerebral cortex B. Thalamus C. Midbrain D. Pons E. Medulla F. Cranial nerves G. Spinal cord H. Spinal nerves

A. Cerebral cortex

71 year old man suffers from progressive losses of memory and concentration. He is increasingly apathetic and unmotivated. He exhibits an awkward gait, having particularly difficulty when encountering stairs or curbs. He recently wrt his pants but seems unconcerned. He does not vomit, and nausea is absent. CSF pressure, assessed by lumbar puncture, is in the high normal range. What is the most likely diagnosis? A. Obstructive hydrocephalus B. Communicating hydrocephalus C. Alzheimer's disease D. Parkinson's disease

A. Communicating hydrocephalus (has presence of cognitive decline, and motor problems, but no vomit so no obvious increase in ICP as CSF pressure is high normal, and apathetic)

Which of the following motor pathways is principally involved in precise voluntary motor activity of the limbs? A. Corticospinal B. Corticobulbar C. Vestibulospinal D. Reticulospinal E. Rubrospinal

A. Corticospinal

A 25-year-old woman comes to the physician because of a 1-month history of seizures. She is prescribed valproic acid which acts by inhibiting the GABA-transaminase enzyme. Which of the following is the most likely function of this enzyme? A. Degradation of GABA B. Synthesis of GABA C. Conversion of glutamate to GABA D. Conversion of glutamate to glutamine E. Conversion of glutamine to glutamate

A. Degradation of GABA

A 61-year-old man suffers from a pulmonary tumor. During the illness, he develops bilateral muscular weakness. When he attempts to squeeze his clinician's fingers, muscular contraction is initially minimal. However, with repeated/sustained effort, he temporarily regains strength. The disease is categorized as immune-inflammatory. Among other therapeutic approaches, the patient receives a potassium channel blocking agent. By which of the mechanisms does the drug most likely restore strength in the patient? A. Enhanced transmitter release B. Suppressed transmitter release C. Speeded axonal conduction D. Slowed axonal conduction E. Increased transmitter synthesis F. Decreased transmitter synthesis

A. Enhanced transmitter release

A 53-year-old man comes to the physician because of a 3-week history of problems hearing from his left ear. He has a 2-year history of Parkinson's disease. The ear is not painful. Which of the following sites is most likely implicated in the hearing loss? A. External acoustic meatus B. Middle ear C. Inner ear D. Cochlear nuclei E. Right primary auditory cortex

A. External acoustic meatus

A 23-year-old man is brought to the emergency room because of a two-day history of weakness in the lower limbs, and a one-day history of difficulty using his hands. Three weeks prior, he had a GI infection from which he fully recovered. Neurological examination shows reduction in muscle strength in the upper and lower limbs. Deep tendon reflexes for the upper limbs are 1+ and 0 for the lower limbs. Nerve conduction studies show conduction velocity of 35m/s. Which of the following diseases is the most likely cause of the signs and symptoms present in this patient? A. Guillain Barre syndrome B. Leprosy C. Lambert-Eaton syndrome D. Myasthenia gravis

A. Guillain Barre syndrome

A. Which symptoms will be seen in a patient who suffered from cell death of cells in the ventral horn of the spinal cord? B. Which types of motor units do exist? C. How is muscle strength regulated?

A. LMN deficits, even if it's at the level of the spinal cord (hypotonia, hyporeflexia, flaccid paralysis, atrophy/weakness, fasciculation, fibrillation, etc.) B. A single alpha motor neuron + innervated extramural fibers C. Firing rate of motor units and the amount of motor units recriuted

A. What is the difference between anterior and lateral corticospinal tracts? B. What are the implications of the decussation of the lateral corticospinal tract?

A. Lateral -> limbs (90% of corticospinal tracts) Anterior -> trunk B. It's where the tract crosses over in the medulla, so if lesion is ABOVE, it's CONTRALATERAL. If it's BELOW lesion, it's IPSILATERAL

A 37-year-old man is brought to the general hospital by an ambulance because of a sudden loss of sensory and motor functions on the right body and right face. An angiogram shows a unilateral loss of cortical blood supply through the middle cerebral artery. On which side is the arterial blockage most likely located? A. Left B. Right

A. Left

A 77-year-old man is brought to the emergency department because of severe headache and nausea for 3 days. An MRI shows a mass on the lateral aspect of his right frontal lobe. Which of the following parts of his body is most likely to show motor deficits? A. Left upper limb B. Left lower limb C. Right upper limb D. Right lower limb

A. Left upper limb

A. What is the blood supply of the precentral gurus and the anterior paracentral lobule? B. What role do these areas play in motor activity? C. What is the importance of the thalamus in relation to motor activity?

A. MCA; ACA B. MCA -> lateral -> motor activity of the upper limbs ACA -> medial -> motor activity of the lower limbs C. Relays/allows communication of other parts of the CNS for regulation of the motor cortex by the basal ganglia and the cerebellum

What do you associate with these structures? A. Malleus B. Mastoid antrum C. Stapes D, greater petrosal nerve E. Prominence of superior semicircular canal F. Jugular build G. Internal carotid artery

A. Malleus (1st pharyngeal arch, shaped like a mallet B. Mastoid antrum (air space, associated with posterior wall, mastoiditis => mastoidectomy) C. Stapes (stirrup, otosclerosis, 1st pharyngeal arch, oval window D. Greater petrosal nerve (preganglionic parasympathetic to lacrimal, damage results in loss of salvation) E. Prominence of superior semicircular canal (contains perilymph, associated with balance function F. Jugular bulb (in jugular fossa) G. Internal carotid artery (goes near anterior wall, has 3 branches: ACA, MCA, PCA)

A scientist is studying how NO is released for: one group o to signal to another at a synapse. Which one of the following is a feature o fit his signaling molecule? A. NO is a diffusible gas B. NO is a peptide neurotransmitter C. NO synthesis requires choline D. NO decreases cGMP levels within target cells E. NO is formed from glutamate

A. NO is a diffusible gas

List some of the major characteristics of the BBB. Why must L-DOPA decarboxylase inhibitor be given at the same time as L-DOPA for treatment of Parkinson's disease?

A. No fenestrations, physical and chemical barrier, endothelium, continuous basement, tight junctions, foot processes of astrocyte, efflux proteins. This is necessary because the brain doesn't have adaptive immunity as the brain lacks the capacity to rebuild so it doesn't want to defend itself in war and would rather not go into war at all so it doesn't have to worry about rebuilding knowing it will be permanently damaged. B. L-DOPA can cross the BBB but it gets quickly degraded from the DOPA carboxylase to dopamine, which as an amine can't cross the BBB, and the endothelial cells near the brain have a lot of DOPA carboxylase. So the inhibitor is given along with Carbidopa

Discuss the clinical signs seensieth lesions to different parts of the motor unit. Use the table it predict the clinical signs that would be expected in A. Poliomyelitis B. Gillian-Barre Syndrome C. Myasthenia Gravis

A. Poliomyelitis (soma; inc. WBC, asymmetric muscle weakness and atrophy, LMN lesion with hyperreflexia and fasciculations/fibrillations) B. Gillian-Barre Syndrome (Schwann cells; acute demyelination polyradiculoneuropathy after respiratory or GI infection, starts from feet up so it's ascending paralysis, ascending muscle weakness) C. Myasthenia Gravis (NMJ/End plate; muscle weakness with continued use, AChR destroyed, dysphasia, diplopia, astronomers face)

During a physical examination, mechanical stimulation is applied to the skin of a patient. The patient reports detecting the stimulus. Which of the following cells and electrical events are critical to the detection of the stimulus? A. Primary afferent; receptor and action potential B. Primary afferent; receptor potential only C. Primary afferent; action potential only D. Primary efferent; receptor and action potential E. Primary efferent; receptor potential only F. Primary efferent; action potential only

A. Primary afferent; receptor and action potential

An accidental rupture of the eardrum goes along with short period of pain. How would that pain be classified? A. Primary otalgia B. Secondary otalgia

A. Primary otalgia

A 22-year-old man is brought to the emergency department because of an acute stab wound on his right back, as illustrated below. Neurological examination shows loss of touch, vibration and proprioception sensations on the right at T4 and below. Axons of which somatosensory neurons are most likely damaged? A. Primary somatosensory neurons B. Secondary somatosensory neurons C. Tertiary somatosensory neurons

A. Primary somatosensory neurons

A 56-year-old man comes to the physician because of a 6-hour history of tingling and clumsiness affecting his right arm and leg. Physical examination shows right-sided loss of proprioception, vibration and touch. Pain and temperature senses are normal bilaterally. Which of the following pathways is most likely dysfunctional? A. Right dorsal column B. Left dorsal column C. Right anterolateral system D. Left anterolateral system E. Right corticospinal system F. Left corticospinal system

A. Right dorsal column

A 53-year-old man comes to the physician because of a 3-month history of progressive weakness with tingling and numbness affecting the left trunk and left lower limb. A functional lesion affecting which of the following lobes best explains the motor manifestations? A. Right frontal B. Left frontal C. Right parietal D. Left parietal E. Right temporal F. Left temporal G. Right occipital H. Left occipital

A. Right frontal

A group of researchers investigates the effects of prenatal cocaine exposure in rats. It is hypothesized that maternal cocaine exposure gene-specifically alters mRNA expression in the embryo. The dysfunctional release of a secreted protein will then cause defective formation of the ventral morphology of the neural tube. Which of the following is most likely under study? A. SHH B. BMP C. BDNF D. Laminin E. Astrotactin F. Reelin

A. SHH

A male neonate is assessed for a significant birth defect characterized by deep sulci (fissures) reaching almost the ventricles. Which of the following terms defines the diagnosis? A. Schizencephaly B. Spina bifida occulta C. Spina bifida D. Anencephaly

A. Schizencephaly

A 24-year-old man comes to the emergency department because of a 2-hour history of pain to his left eye. He received a blow to the left eye while gardening 4 hours ago. Physical examination shows a puncture wound in the sclera. Which of the following best characterizes the affected structure? A. Site of attachment of extraocular muscles B. Forms the basement membrane for the retinal pigmented layer C. Made up of ten (10) layers D. Contributes to the color of the eye E. Produces zonule fibers F. Composed of crystalline proteins

A. Site of attachment of extraocular muscles

Following sand blast, a 25 year old man experiences pain arising from the skin on his back and shoulders. The skin is red and tender consequent to over-exposure to the sun. Which cutaneous receptor mediate the discomfort? Are they rapidly or slowly adapting?

A. Temperature —> thermoreceptors —> free nerve endings B. Slowly adapting because it's a burning pain

Vascular lesions can interrupt the pain pathway at different levels. A. What fibers carry pain sensation from the face? B. From which level up could pain in the body and face be affected? C. Describe an example of a vascular lesion that affects pain unilaterally on the same side of he body, as if the face, and discuss the link between the deficient artery and the damaged structure(s) carrying the pain signal, and the deficit(s) in pain perception experienced by the patient

A. Trigeminal B. (GO BACK TO REVIEW:BLOOD SUPPLY OF THE MEDULLA IN LECTURE)

A 77-year-old man is brought to the emergency department because of severe headache and nausea for 3 days. An MRI shows a mass on the lateral aspect of his left frontal lobe. Which of the following parts of his body is most likely to show motor deficits? A. Upper limb B. Lower limb

A. Upper limb

At what age do people usually develop retinal detachment?

Age 40-70, but more in their 50s

What is the role of the cerebellar peduncles?

Allows for communication between the cerebellum and brainstem -superior cerebellar (more efferent than afferent) -middle cerebellar (purely afferent) -inferior cerebellar/VIII (more afferent than efferent)

How does the angular vein connect with the cavernous sinus and the pterygoid plexus? What is the significance of knowing this?

Angular vein connect via ophthalmic v. which connects to the other pterygoid plexus and branches from the canervous sinus. This can be a spread of infection (cavernous sinus infection and/or thrombosis) and can have ocular and or sensory/motor manifestation Cavernous has (CN 3, 4, V1, V2, 6)

Info arising from hair cells located closest to the helicotrema is processed by cells located on which region of the primary auditory cortex?

Anterior part of your primary auditory cortex

Which is the smallest part of the temporal bone visible in a submental view? A. Squamous B. Petrous C. Tympanic D. Styloid

B. Petrous

Review the functional structure of the cerebellum.

At the center (medial) = vermis Lateral = limbs Anterior lobe (spinocerebellum - sensory feedback) posterior lobes (cerebrocerebellum - motor and sensory feedback for precise movement and skills) Flocculonodular lobe (vestibulocerebellum - afferent from vestibular apparatus such as balance or eye movement)

A 35-year-old man comes to the emergency department because of a 3-week history of neck pain, right-sided arm pain and weakness in right hand grip. The pain radiates down his arm to his hand. An MRI shows a herniated disk compressing the spinal nerve exiting through the intervertebral foramen between the 7th cervical vertebra and 1st thoracic vertebra. Which of the following neurological findings is most likely to be seen on deep tendon reflex testing of the affected limb in this patient? A. Biceps reflex 1+, Triceps reflex 1+ B. Biceps Reflex 2+ , Triceps reflex 1+ C. Biceps Reflex 2+, Triceps reflex 2+ D. Biceps Reflex 2+, Triceps reflex 4+ E. Biceps Reflex 4+, Triceps reflex 4+ F. Biceps Reflex 4+, Triceps reflex 1+

B. Biceps Reflex 2+ , Triceps reflex 1+ -nerve in question is C8 which affects the triceps, so biceps would be normal

Which of the following statements about the incus is matching best? A. Well anchored B. Can dislocate and cause conductive hearing loss C. 2nd arch derivative D. 3rd arch derivative E. Chorda tympani travels between incus and the stapes in the middle ear F. The body of the incus can sometimes be seen parallel to the malleus on otoscopy

B. Can dislocate and cause conductive hearing loss

A 43-year-old man is brought to the emergency department because of a sudden loss of motor function. Imaging suggests a Medial Medullary Syndrome. Loss of blood supply is illustrated by the darkened area in the schematic drawing below. Neurological examination is performed.On which side has the patient most likely lost touch, vibration, and proprioception sensations? A. Ipsilateral B. Contralateral

B. Contralateral

Which of the following effects on movement is most likely in a patient with dopamine deficiency in the substantia nigra? A. Increase B. Decrease C. None, increase and decrease cancel each other out

B. Decrease

A 36-year-old man comes to the physician because of a 2-year history of laryngeal dystonia. He receives injections of botulinum neurotoxin into the overactive laryngeal muscles. Within a few days, his voice quality improves, and he is able to speak fluently. Which of the following mechanisms of action is most likely responsible for the clinical improvement? A. Blocking calcium channels B. Decreasing acetylcholine release C. Increasing acetylcholine release D. Inhibition of acetylcholinesterase E. Blocking of acetylcholine receptors

B. Decreasing acetylcholine release - were messing with transmitter release of LMN which uses ACh

A researcher assesses substances for their abilities to cross the blood brain barrier. She plots the substances against their Extraction Ratios as a function of the Oil/Water Partition Coefficients. Based on this plot, which of the following substances is most likely able to cross the blood brain barrier? A. Mannitol, Dopa, Glucose B. Diazepam, Glucose, Heroin C. Dopamine, Penicillin, Mannitol D. Nicotine, Sodium, Morphine E. Ethanol, Dopa, Morphine

B. Diazepam, Glucose, Heroin

A 67-year-old man is brought to the physician because of a 1-year history of difficulties moving his limbs. Neurological examination shows rigidity, resting tremor, slow gait and an expressionless face. Which of the following neurotransmitters is most likely deficient? A. Acetylcholine B. Dopamine C. Glutamate D. Gamma aminobutyric acid E. Serotonin

B. Dopamine

A 50 year old man has pyridoxine deficiency. Which of the following reactions is most likely impaired in this patient? A. Formation of glutamine rom glutamate B. Formation of GABA from glutamate C. Synthesis of acetylcholine D. Synthesis of nitric oxide from arginine E. Formation of methionine form homocysteine

B. Formation of GABA from glutamate

Which of the following transmitters typically acts in an inhibitory fashion following its release from small CNS interneurons? A. Glutamate B. GABA C. Acetylcholine D. Dopamine E. Norepinephrine F. Serotonin

B. GABA

A 57-year-old man comes to the physician because of a 3-month history of tremors. Physical examination shows resting tremors in the right hand. He writes slowly, and the muscles of his right arm are rigid. Carbidopa-levodopa is prescribed and it is expected to normalize movement by increasing synaptic availability of a neurotransmitter in the brain regions indicated in the neuroimage. Levels of which of the following metabolites would most likely be elevated in the urine of this patient following treatment? A. VMA B. HVA C. 5-HIAA D. SAM E. BH4

B. HVA

A 77-year-old man is brought to the emergency department because of severe headache and nausea for 3 days. An MRI shows a mass on the lateral aspect of his left frontal lobe. Which of the following signs is most likely present in this patient? A. Hyporeflexia B. Hyperreflexia

B. Hyperreflexia

A 60-year-old man is brought to the physician because of tremors of the right hand for 3 months. Physical examination show rigidity of the right upper limb and resting tremor of the right hand. He has reduced facial expressions and monotonous speech. Which of the following terms best classifies this disease? A. Hyperkinesia B. Hypokinesia

B. Hypokinesia

Swelling of the optic nerve would likely cause the perceptual blind spot to A. Decrease B. Increase C. Stay the same

B. Increase in Papilledema d/t increased ICP

A 33-year-old woman is brought to the emergency department because of a 3-hour history of unconsciousness. Her spouse says that she fell from a ladder and struck her head 1 day ago. Imaging shows a epidural hemorrhage. She dies before surgical intervention is possible. Between which of the following tissue layers is the blood most likely accumulating? A. Outer skull; periosteal dura B. Inner skull; periosteal dura C. Periosteal dura; meningeal dura D. Meningeal dura; arachnoid E. Arachnoid; pia F. Pia; brain

B. Inner skull; periosteal dura

A 14 year old boy is brought to the physician by his mother for a follow up exam. He received chemical burns to his right eye 1 week ago during a science experiment at his schools lab. He complai;s of intense pains in his right eye, PE shows redness of the conjunctiva and decreased visual acuity in his right eye, the physician suspects that the injury may have been extensive enough to affect the corneo-limbal stem cells. If this suspicion is accurate, which of the following can be a consequence of the affected tissue? A. Blockage of the veins at the canal of Schlemm B. Poor regenerative capacity of the corneal epithelium C. Loss of opacification of the lens D. Dehydration of the vitreous body E. Destruction of corneal epithelium

B. Poor regenerative capacity of the corneal epithelium —> leads to glaucoma

A 33-year-old woman is brought to the emergency department because of a sudden loss of motor function of her left leg. Neurological examination shows loss of touch, vibration, proprioception, pain and temperature sensations in the same area. Which artery is most likely impacted? A. Left anterior cerebral B. Right anterior cerebral C. Left middle cerebral D. Right middle cerebral E. Left posterior cerebral F. Right posterior cerebral

B. Right anterior cerebral

68 year old woman with history of HTN comes to the ED because of sudden onset of nausea and vomiting. PE she's slurred speech and dye metric on finger to nose testing on the right side. Her gait is normal. Where is the lesion most likely located? A. Right cerebral peduncle B. Right cerebellar hemisphere C. Right subthalamic nucleus D. Left cerebellar hemisphere E. Left cerebral peduncle F. Left subthalamic nucleus G. Vermis

B. Right cerebellar hemisphere (Know difference between cerebral and cerebellar peduncles)

*****A 24-year-old man is brought to the physician because of a 3-day history of difficulty controlling his right hand and foot. Neurological examination shows deficits in precise movements of the right limbs. Imaging studies show an infarction of the cerebellum. Which of the following locations is most likely lesioned? A. Right anterior lobe B. Right posterior lobe C. Right flocculonodular lobe D. Left anterior lobe E. Left posterior lobe F. Left flocculonodular lobe

B. Right posterior lobe

A 31 year old man comes to the physician's office complaining of pain in his left ear for 3 weeks. The patient has a history of frequent ear infections and eustachian tube dysfunction. Otoscopy shows a white mass surrounding the malleus. A large perforation of the tympanic membrane is also seen (please see the attached image). Prolonged eustachian dysfunction may lead to retraction of the tympanic membrane. The mass is most likely composed of which of the following substances? A. Pseudostratified epithelium B. Stratified squamous keratinized epithelium C. Simple cuboidal epithelium D. Cholesterol E. Stratified squamous non-keratinized epithelium

B. Stratified squamous keratinized epithelium

A 31-year-old man is brought to the emergency department because of a 2-hour history of a penetrating head wound. A sharp fragment of metal penetrated the skull and entered his brain. A population of glial cells will transiently aggregate in the damaged nervous tissue. Which of the following activities best characterizes these glial cells? A. Ion-buffering B. Surveilling and phagocytosing C. Electrically insulating D. CSF-producing E. Axon-guiding

B. Surveilling and phagocytosing

An 11-year-old girl comes to the physician because of abnormal fidgety movement of the body, fever and joint pain for 2 days. She has a history of a sore throat 4 weeks ago. Physical examination shows rapid, jerky, irregular, and involuntary movements. Laboratory studies show positive antistreptolysin O titer, and a diagnosis of rheumatic fever is made. Which of the following diagnoses is most likely? A. Parkinson's disease B. Sydenham's disease C. Huntington's disease D. Hemiballismus

B. Sydenham's disease

A 50-year-old man is bought to the emergency room by the police. He is incoherent, malnourished and appears unkempt. He is recognized by the staff because he has been admitted previously for alcohol related issues. Which lab test would most likely be used to reveal thiamin PP (Vit B1) deficiency in this patient? A. FIGlu assay B. Transketolase assay C. Pyruvate DH assay D. Alpha KG DH assay E. Pyruvate carboxylase assay

B. Transketolase assay

A 21 year old man is injured during a fight in a college bar. A shard of glass entered his forearm, transecting the radial nerve, instantly yielding numbness and weakness of the distal extremity, over many days pathological changes to the nervous system arose secondary to the traumatic event. Identify the process whereby the distal ends of the transacted axons degenerated A. Chromatolysis B. Wallerian degeneration C. Retrograde transneural degeneration D. Anterograde transneural degeneration

B. Wallerian degeneration -chromatomysis she's change in staining of cells in cell bodies, and it changes its appearance usually associated with apoptosis. -So what we have in comparison to the other ones, Wallerian degeneration is the degeneration of the axon severed and every thing downstream from the dissection is lost (action, myelin sheath synaptic ending all gone), but still leaves us with cell body and a small section intact of the axon. The axon can also degenerate with the cell body showing an indication of myolysis, then apoptosis for cell death. -retrograde goes even further so the previous neuron (presynaptic) will be damaged -anterograde has postsynaptic neuron losing is innervation d/t degeneration from the distal end

Why would the brain be such an exception from the normal range of immune functions

Because of the BBB. The tight junction will only allow so much to pass through, we don't have gap junctions and we do have cilia (all from the ependymal cells)

Why are receptor or post synaptic potentials graded?

Because they are not always firing because there's a specific threshold that needs to be met for an AP. You don't want any stimulus activating AP because it can cause and reaction to occur, so there needs to be a sufficient trigger that regulates activation of the AP

A 55-year-old man comes to the physician because of a 4-month history of weakness of the hands, particularly the thumb. He also has trouble swallowing. Physical examination shows fasciculations of the hands. Which of the following components of the motor unit is most likely lesioned? A. Axonal terminal B. Muscle cell C. Neuronal cell body D. Schwann cell E. ACh receptor

C. Neuronal cell body

The image illustrates left spinal cord hemisection. Concerning the right side of the body, from which level(s) in relation to the lesion will pain and temperature senses be lost? A. Above the level of the lesion B. At the level of the lesion C. Below the level of the lesion

C. Below the level of the lesion

A 31-year-old man comes to the physician because of a 5-year history of recurrent sensory disturbances, muscle cramps of the thighs and tinnitus. He is diagnosed with a hereditary neuropathy tied to substitution of aspartate for glycine. Which of the following patterns of findings is most likely in this patient? A. B. C. D. Reduced tendon reflex O O Optic neuritis. O O Foot weakness. O O O Normal cerebral MRI. O O O Cognitive impairment. O O

C. C - Charcot Marie Tooth

A 27-year-old woman comes to the physician because of numbness and tingling of her left index finger and thumb for 1 week. She feels pain in the same area, which gets worse at night. Neurological examination shows a weak thumb and a reduction in grip strength. Nerve conduction velocity of the median nerve at the wrist is 40 m/s on the left and 55 m/s on the right. What is the most likely diagnosis? A. Diabetic polyneuropathy B. Lambert-Eaton syndrome C. Carpal tunnel syndrome D. Myasthenia gravis

C. Carpal tunnel syndrome

A 68 year old woman comes to the physician because of increasing fatigue that has progressed over the past year. Lab studies show megaloblastic anemia. She also has forgetfulness, memory impairment, and mood changes. A bonus of B12 resulted in improvement over a few days. Prior to the injection of B12, which of the following molecules was most likely elevated in this patient? A. SAM B. Cysteine C. Methylmalonyl D. NADPH E. Formyl-THF F. Epinephrine

C. Methylmalonate

A 25-year old woman comes to the physician because of a 3-month history of worsening vision. She also reports increasing weakness when walking and frequent numbness and tingling sensations in the extremities. What is the most likely cause of these problems? A. Tumor near optic nerve B. Guillain-Barré syndrome C. Multiple Sclerosis D. K+ deficiency

C. Multiple Sclerosis

A 29-year-old man comes to the ophthalmologist because of double vision. During primary gaze, his left eye is abnormally adducted. Which of the following nerves is most likely lesioned? A. Left CN III B. Left CN IV C. Left CN VI D. Right CN III E. Right CN IV F. Right CN VI

C.

A 40-year-old man dies following a prolonged illness which became progressively worse over the past 6 months. He was confined to his bed and could not perform his daily activities. Autopsy findings show Creutzfeldt-Jakob disease. Which of the following autopsy findings is most likely present in his brain? A. Deposition of bilirubin in basal ganglia B. Increased serotonin formation in APUD cells C. Accumulation of prion proteins with beta-pleated sheets D. Accumulation of beta-amyloid precursor protein E. Lysosomal accumulation of sphingomyelin F. Accumulation of polyglutamine rich huntingtin protein

C. Accumulation of prion proteins with beta-pleated sheets

A 27-year-old woman comes to the physician for a follow-up examination. Two days earlier, she had been diagnosed with a vascular lesion involving her right optic tract, as illustrated in the MRI below. Which of the following responses during pupillary light reflex testing are most likely? A. Absence of pupillary constriction on the left B. Absence of pupillary constriction on the right C. Diminished pupillary constriction in both eyes D. Full pupillary constriction in both eyes

C. Diminished pupillary constriction in both eyes

A 65- year-old man is admitted to the neurosurgery unit for surgical ablation of the major output nucleus of the basal ganglia. What is the most likely effect of this surgery on the thalamus? A. Excitation B. Inhibition C. Disinhibition

C. Disinhibition

A 15-year-old girl is brought to the physician because of loss of balance, increasing difficulties in walking and moving her arms. Examination shows weakness and loss of sensation in her feet. Additionally, her distal muscles showed signs of wasting. DNA tests confirmed an inherited genetic mutation. Which of the following is most likely causing her condition? A. Decreased Laminin synthesis B. Decreased Netrin levels C. Dysfunctional PMP-22 expression D. Too much retinoic acid E. Lack of Laminin

C. Dysfunctional PMP-22 expression

A 22-year-old woman comes to the physician because of a 24-hour history of sensory disturbances that began after striking her head. An injury overlapping with the encircled cortical area is suspected. Which of the following sensory modalities is most likely impaired? A. Touch B. Vision C. Hearing D. Smell E. Taste F. Pain

C. Hearing

Which disease does syringomyelia most closely resemble? A. Periventricular hemorrhage B. Subarachnoid hemorrhage C. Hydrocephalus D. Huntington disease

C. Hydrocephalus (GO BACK TO PREVIOUS RECORDING FROM EARLIER!)

What acoustic phenomenon is experienced by patients with Bell's Palsy? A. Tinnitus B. Deafness C. Hyperacusis D. Presbycusis

C. Hyperacusis -more vibration due to paralysis

Which of the following sensory fibers innervates the dynamic nuclear bag fiber? A. III B. IV C. Ia D. Ib E. II

C. Ia

A 43-year-old man is brought to the emergency department because of a deep injury on the face. He had cut himself while shaving, but he did not feel any pain. As far as he remembers, his pain perception was normal the night before. Imaging suggests a lesion of the posterior inferior cerebellar artery (PICA), as illustrated in the schematic drawing below. Neurological examination is performed. Where is his loss of pain sensations most likely located? A. Ipsilateral face and body B. Contralateral face and body C. Ipsilateral face and contralateral body D. Contralateral face and ipsilateral body

C. Ipsilateral face and contralateral body

A 29-year-old man comes to the ophthalmologist because of double vision. During primary gaze, his left eye is abnormally adducted. Which of the following extraocular muscles of his left eye has most likely lost its innervation? A. Inferior oblique B. Inferior rectus C. Lateral rectus D. Medial rectus E. Superior oblique F. Superior rectus

C. Lateral rectus

A 64-year-old man comes to the physician because of a 2-day history of problems with movement and slurred speech. He has a 10-year history of hypertension. Physical examination shows dysmetria in the left hand and foot. Which of the following cerebellar sites is most likely damaged? A. Vermis B. Right lateral hemisphere C. Left lateral hemisphere D. Right fastigial nucleus E. Left fastigial nucleus F. Right flocculus G. Left flocculus

C. Left lateral hemisphere - problem with precision, so relates with cerebral cerebellum = lateral left hemisphere, and cerebellum is IPSILATERAL to motor control

A 33-year-old woman comes to the physician because of a 3-day history of problems moving. Neurological examination shows left-sided intention tremor, past-pointing and dysdiadochokinesia. Imaging studies show a lesion affecting the left ventrolateral surface of the pons, overlapping with the transverse fibers. Which of the following cerebellar peduncles is most likely affected? A. Left inferior B. Right inferior C. Left middle D. Right middle E. Left superior F. Right superior

C. Left middle

A 36-year-old women comes to the physician because of a migraine attack. When she was brushing her hair in the morning, the pain was almost unbearable. Which of the following changes in receptor thresholds is most likely responsible for the pain induced by brushing her hair? A. Lowered threshold of touch receptors B. Raised threshold of touch receptors C. Lowered threshold of nociceptors D. Raised threshold of nociceptors

C. Lowered threshold of nociceptors

A 45-year-old man is brought to the physician because of problems using his hands. Neurological examination shows dysmetria, dysdiadochokinesia and an intention tremor. Which of the following structures is most likely the embryological precursor to the lesioned structure? A. Telencephalon B. Myelencephalon C. Metencephalon D. Mesencephalon E. Diencephalon

C. Metencephalon

A 22-year-old healthy man has been starving for 4 days. The majority of the energy for his brain function comes from the oxidation of ketone bodies, but his brain still has an absolute need for glucose. Which of the following function is this glucose most likely required for? A. Formation of S-adenosylmethionine B. Anaerobic ATP production C. Neurotransmitter biosynthesis D. The resting membrane potential E. Formation of vitamin B1

C. Neurotransmitter biosynthesis

A 73-year-old man suffers from memory and visuo-spatial deficits. He receives a diagnosis of Alzheimer's disease. Which of the following nuclei is most likely degenerating? A. Substantia nigra B. Nucleus raphe magnus C. Nucleus basalis of Meynert D. Locus coeruleus

C. Nucleus basalis of Meynert

Which of the following are correct statements? A. Glutamate is formed form glutamine by glutamate dehydrogenase B. Glutaminase forms GABA from glutamate C. PLP is required for glutamate decarboxylase D. Glutamate decarboxylase forms GABA from glutamine

C. PLP is required for glutamate decarboxylase

A researcher studies the histology of the vascular endothelium in regions of the brain that lack a blood brain barrier. Through which of the following endothelium-related routes do substances most likely pass to enter the brain from the blood? A. Intracellular B. Intercellular C. Paracellular D. Circumcellular E. Epicellular

C. Paracellular

A 47-year-old man comes to the physician because of a 6-week history of stabbing pain emanating from the back of the tongue and adjacent throat on the right. The bursts of pain were initially infrequent, but now eating or talking consistently provoke the pain. Which of the following lobes most likely mediates the painful episodes? A. Frontal B. Occipital C. Parietal D. Temporal E. Limbic F. Insular

C. Parietal

An 81 year old woman is brought to the emergency department because of sudden onset dimming of vision. Assessment of the visual system shows that she is blind to the right half of visual space but with preservation of he center of the visual field. Cortical infarction is suspected. Which artery is most likely implicated in the infarction? A. Anterior cerebral artery B. Middle cerebral artery C. Posterior cerebral artery

C. Posterior cerebral artery

After a diagnostic lumbar puncture, a 27-year-old woman develops nausea and enlarged pupils. In which of the following intracranial fossae are structural anomalies evident? A. Anterior B. Middle C. Posterior

C. Posterior; Part of the cerebellum gets sucked into the brainstem, specifically the medulla oblongata, and this is cerebellu par tonsils very low, under foramen magnum (Arnold chiari)

By weight (and at rest) the brain consumes more ATP than any other component of the body. Which of the following activities is estimated to be responsible for the majority of this energy consumption? A. Elongation of fatty acids B. Transamination of glutamate/a-KG C. Primary active transporter - Na/K ATPase D. Synthesis of methionine from homocysteine E. Uptake of ketone bodies during fasting state

C. Primary active transporter - Na/K ATPase

A 32-year-old man is brought to the physician by his wife because of a 1-month history of loss of memory. He has a 1-year history of depression and excessive alcohol consumption. His food habits are erratic and unhealthy. Which of the following enzymatic activities is most likely impaired in this patient? A. Acetylcholine esterase B. Glutamate dehydrogenase C. Pyruvate dehydrogenase D. Methylmalonyl CoA mutase E. Glutamate decarboxylase F. Histidine decarboxylase

C. Pyruvate dehydrogenase -chronic alcoholism = Wernicke Kosakoff syndrome using thiamine

A 36-year-old man is brought to the emergency department because of "double vision" following an automobile accident 20-minutes ago. He was hit in the right eye with the airbag as it deployed. His vital signs are all within normal limits and he is conscious. Physical examination shows discoloration of the orbit of the right eye and blurred vision when focusing on near objects. It is suspected that he has disruption of the ciliary body with paralysis of the ciliary muscles of the right eye (cycloplegia). This is a common injury following blunt trauma to the eyes. Which of the following is an additional finding in this patient specific to the sustained? A. Loss of lens transparency B. Separation of the RPE from the outer segments of rods and cones C. Reduction in aqueous humor production D. Destruction of the rods E. Decreased pupillary light reflex

C. Reduction in aqueous humor production

This is an image of a 26-year-old man's eye who is about to go for LASIK (laser-assisted in situ keratomileusis) surgery to correct refraction. Which of the following statements is most likely relevant to this case: A. LASIK surgery of the cornea will aggravate this condition B. This is the result of an acquired condition due to infection in the anterior chamber, healed with adhesions C. Result of failure of retinal fissure closure D. This pupil size is fixed E. A late development from contracting measles in the 16th week of pregnancy

C. Result of failure of retinal fissure closure

A 75-year-old man is brought to the emergency room because of a 2-hour history of sudden-onset immobility of his left leg. Neurological examination shows extension of the great toe and fanning of the other toes when the sole of his left foot is briskly stroked. His knee jerk and ankle jerk reflexes are 2+ on the right and 3+ on the left. There are no sensory deficits. Imaging studies will most likely reveal a lesion in which of the following structures? A. Right ventrolateral precentral gyrus B. Left ventrolateral precentral gyrus C. Right anterior paracentral lobule D. Left anterior paracentral lobule E. Right spinal anterior horn F. Left spinal anterior horn

C. Right anterior paracentral lobule

A 61-year-old woman is brought to the emergency department because of a collapse in her home. She has left hemiparesis, and her eyes deviate to the left. At which of the following sites is the lesion most likely located? A. Right internal capsule B. Left internal capsule C. Right pons D. Left pons Right. Left Biceps. 1+/3+ Triceps. 2+/3+ Patellar 2+/3+ Ankle 2+/4+

C. Right pons -eyes drift to the left so probably something wrong with the right hemisphere and you can't move your eyes to the right. Left hemiparesis = UMN to the left. If you damage the r frontal lobe the eyeballs can't look left so they look back right, but here the injury is on the right side of the brain and the eyes drifted to the left. Left paramedial pontine nucleus that drives the eyes to the left

A 36 year old man comes to the ED because of an 18 hour history of weakness of lower limbs. He recovered 14 days ago from a GI bacterial infection. The weakness began in his feet and ascended to the hips. Neurological exam shows lower motor neuron signs in the lower limbs. Nerve conduction studies show slowed conduction velocities of the affected peripheral motor nerves. Which of the following parts of the motor unit is most likely affected? A. Soma B. Axon C. Schwann cells D. Muscle fibers E. Neuromuscular junction

C. Schwann cells

A 45-year-old woman comes to the physician for a follow-up examination. She has a 6-month history of depression. She is prescribed tranylcypromine which is an inhibitor of the enzyme monoamine oxidase. Levels of which of the following neurotransmitters is most likely increased following administration of this drug? A. Glutamate B. GABA C. Serotonin D. Acetylcholine E. Histamine

C. Serotonin

A 54-year-old man comes to the physician because of a 6-hour history of numbness affecting the left arm and leg. Neurological testing shows loss of somatosensation for the left arm and leg. Sensations from the face are bilaterally normal. Which of the following structures is most likely injured? A. Precentral gyrus B. Crus cerebri C. Thalamus D. Pyramid E. Olive F. Cerebellum

C. Thalamus

A 42-year-old woman comes to the physician because of drooping of the left corner of her mouth. Her left nasolabial fold is flat. Which of the following corneal reflex findings would be consistent with Bell's palsy on the left? A. No response when touching the left cornea B. No response when touching the right cornea C. The left eye never blinks D. The right eye never blinks

C. The left eye never blinks

Damage to which of the following structures yields hypotonia (select all that apply)? A. Spinal lateral white column B. Nucleus gracilis C. Trochlear nucleus D. Ventral gray horn E. Internal capsule F. Median nerve G. Medial lemniscus

C. Trochlear nerve D. Ventral gray horn F. Median nerve

Brown-Sequard syndrome involves damage to which of the following (select one answer only)? A. Upper motor neurons B. Lower motor neurons C. Upper and lower motor neurons

C. Upper and lower motor neurons

During caloric testing with warm water, the following pattern of eye movements is observed. Application to the left ear causes a slow drift of the eyes to the right. Application to the right ear causes a slow drift of the eyes to the left. Which structures are most likely mediating the observed movement of the eyes? A. Frontal eye fields B. Occipital eye fields C. Vestibular nuclei D. Olivary nuclei

C. Vestibular nuclei

A 29 year old woman comes to her physician because of nausea, postural instability and falls for 2 weeks. Three months ago, she had bariatric surgery. Neurological exam shows nystagmus and ataxic gait. Which of the following is most likely that cause of the patients symptoms? A. Alcoholic cerebellar degeneration B. Cerebellar stroke C. Vitamin B deficiency D. Louis Bar syndrome

C. Vitamin B deficiency

Which type of cerebral white matter transfers information between the left and right occipital lobes? What are the other fibers?

Commisssural fibers (ex. Corpus callosum) is a white fiber that connects one side of the CNS with the other by crossing over in the medulla oblongata Projection fibers connect the cortical and subcritical nuclei, while association fibers connects two regions within one hemisphere

During Weber test, sound lateralizes to the right, what are the most two plausible diagnoses to be considered?

Conductive hearing loss to the right and sensorineural hearing loss to the left

In aging, hair cells tuned to high frequency noise may degenerate. Where are the corresponding hair cells located in relation to the oval window?

Corti of the cochlea, where hair cells most sensitive to high frequency are located

A 31-year-old man is brought to the physician by his wife because of a 9-month history of numbness affecting the upper limbs and trunk. Imaging studies show a fluid-filled lesion affecting the upper spinal cord. Which of the following anatomical planes is most likely aligned with the long axis of the fluid-filled lesion? A. Medial-lateral B. Dorsal-ventral C. Anterior-posterior D. Rostral-caudal E. Ventral-lateral

D. Rostral-caudal

MRI of a 54-year-old female shows a pituitary tumour that appears to be uniformly compressing the midline of the optic chiasm. Which of the following terms best applies to the likely visual field defect? A. Left homonymous hemianopia B. Right homonymous hemianopia C. Binasal hemianopia D. Bitemporal hemianopia

D. Bitemporal hemianopia

A 9-year-old boy is brought to the physician because of a 3-month history of intermittent nausea and vomiting, headache, and poor balance. Neurological examination shows truncal ataxia, hypotonia and dysmetria. Laboratory studies are within normal limits and no cutaneous lesions are seen. Which of the following diagnoses is most likely? A. Louis Bar Syndrome B. Malnutrition C. Cerebellar stroke D. Cerebellar tumor

D. Cerebellar tumor

A man riding a bicycle blinks in reaction to a flying insect approaching one of his eyes. Which of the following combinations best relates to the synaptic contacts and receptors types mediating the protective reflexive response? A. Electrical; metabotropic B. Electrical; ionotropic C. Chemical; metabotropic D. Chemical; ionotropic

D. Chemical; ionotropic B- ionotropic receptors aren't associated with electrical synapses

A 22 year old woman comes to her physician complaining of fatigue which started 4 months ago, she explained that her symptoms were worse in the evening than in the morning. Exam revealed ptosis of both eyelids. When she was asked to perform repetitive movements her strength decreased. A Tensilon test was ordered and she was diagnosed with an autoimmune disease directed against the Nicotinic acetylcholine receptor. Which of the following is the precursor for the neurotransmitter which normally binds to the affected receptor? A. Pyruvate acid B. Choline acetyltransferase C. Acetyl CoA D. Choline E. Acetylcholinesterase

D. Choline- ACh is also metabolized back into choline in the synaptic cleft, which undergoes uptake. It can be used again in the synthesis of further Ach

A 32-year-old woman comes to the physician because of a 1-day history of a discharge from her right eye and a 2-day history of itching and redness of the same eye. Physical examination shows increased redness (as shown in the attached image) and a yellow, viscous discharge in her right eye. Which of the following best characterizes this condition? A. Chalazion B. Iritis C. Stye D. Conjunctivitis E. Cataract

D. Conjunctivitis

A 75-year old man undergoes routine testing of eye movements. During lateral gaze to the right, which of the following choices best represents the primary activation pattern of cranial nerves? A. A B. B C. C D. D E. E F. F

D. D

A 63-year-old man is brought to the emergency department by his wife because of a 4-hour history of difficulty opening his right eye. He has a 15-year history of diabetes. His temperature is 37.8 C (100.1F), pulse is 68/min, blood pressure is 132/83 mmHg and respirations are 17/min. Physical examination shows complete right upper lid ptosis. Which of the following additional signs will most likely be found on physical examination? A. Complete left upper lid ptosis B. Loss of the light reflex in the left eye C. Loss of tears in the right eye D. Dilation of right pupil E. Loss of sensation of the right eyeball

D. Dilation of right pupil

A patient receives an opioid for pain relief. In addition to its analgesic effects, the drug induces euphoria by exciting cells of the nucleus accumbens. Which of the following transmitters undergoes increased release, thereby mediating the rewarding properties of the opioids? A. Glutamate B. GABA C. Acetylcholine D. Dopamine E. Norepinephrine F. Serotonin

D. Dopamine -nucleus accumbens is where dopamine is concentrated

A 65-year-old man comes to the physician because of a 2-week history of difficulty moving his right upper limb. He has a mild aching pain in the right shoulder. There is no loss of sensations (numbness). Neurological examination shows wasting and fasciculations in the right upper limb. Reflexes are 1+ in the right upper limb and 2+ in all other limbs. Which of the signs in this patient is a negative manifestation of a peripheral nervous system disease? A. Fasciculations B. Paresthesia C. Pain D. Hyporeflexia

D. Hyporeflexia

Which of the following sensory fibers is sensitive to muscle tension? A. III B. IV C. Ia D. Ib E. II

D. Ib

A 65 year old man comes to the ED because of a sudden onset of flailing movements of the left upper limb imaging studies reveal a small acute right subthalamic infarction. Which of the following changes is most likely in this patient. A. Decreased cortical excitation B. Decreased activity of the direct pathway C. Increased activity of the indirect pathway D. Increased thalamic disinhibition

D. Increased thalamic disinhibition (stimulating thalamus) -Hemiballismus: hyperkinetic disorder

Which of the following arteries passes through a vein? A. Anterior cerebral B. Middle cerebral C. Posterior cerebral D. Internal carotid E. External carotid F. Vertebral

D. Internal carotid Rupture or leak in cavernous sinus, you can get back flow and can develop a black eye

A 25-year-old man is brought to the emergency department after being found starving for 9 days. He was lost in the woods and he had access to water but no food. Which of the following molecules is oxidized in the brain of this patient to provide the majority of the ATP during this period of starvation? A. Formimino-glutamate B. Branched chain amino acids C. Dietary essential fatty acids D. Ketone bodies E. Glucose F. Fatty acids released from adipose

D. Ketone bodies

A dendrite would exhibit less decrement of a graded electrical potential over distance if it had a ......... diameter or a ......... trans-membrane electrical resistance. A. Smaller, smaller B. Smaller, larger C. Larger, smaller D. Larger, larger

D. Larger, larger

A 22-year-old woman comes to the physician because of a 3-week history of clumsiness involving the left side of the body. Neurological examination shows left dysdiadochokinesia, past-pointing and intention tremor. Imaging studies show an inflammatory lesion to the white matter of the cerebellum. Which of the following cerebellar lobes is most likely lesioned? A. Right anterior B. Left anterior C. Right posterior D. Left posterior E. Right flocculonodular F. Left flocculonodular

D. Left posterior -left cerebellar problem, and the issue is precision so they can't pronate the hand, failed the finger to nose test and have tremors

Compared to plasma, CSF exhibits which of the following? A. Higher glucose content B. Higher protein content C. Lower osmolarity D. Lower pH E. Lower Na+ conten

D. Lower pH

A 28-year-old woman comes to the physician because of a 4-week history of episodes of alternating right and left sided weakness of her lower limbs. She also says she had vision impairment a month earlier which improved overtime. Neurologic examination shows mild residual weakness in the right and left lower extremities. MRI is done and a diagnosis of Multiple sclerosis is made. Which of the following best describes the mode of inheritance of this disorder? A. Multifactorial, males more commonly affected B. Autosomal dominant with gender bias C. Autosomal recessive disorder D. Multifactorial, females more commonly affected E. Mitochondrial disorder with heteroplasmy F. Autosomal dominant, triplet repeat disorder

D. Multifactorial, females more commonly affected

Which of the following structures is involved in monitoring changes in muscle length? A. Golgi tendon organ B. Merkle cell ending C. Motor unit D. Muscle spindle E. Pacinian corpuscle

D. Muscle spindle

A 21-year-old man is brought to the emergency department because of a 1-hour history of a penetrating wound to the back. Physical examination shows initial resistance when the arm is passively extended. With sustained force applied to the limb, the heavily contracted muscle suddenly relaxes. Which of the following receptors is most likely involved in the biphasic response (i.e., contraction followed by relaxation of the stretched muscle)? (Answer: initial resistance/sudden relaxation) A. Golgi tendon organs/free nerve endings B. Golgi tendon organs/muscle spindles C. Muscle spindles/free nerve endings D. Muscle spindles/Golgi tendon organs

D. Muscle spindles/Golgi tendon organs

A 75-year-old man is brought to the physician by his wife because of a 2-month history of inability to carry out everyday tasks. He forgets many recent conversations with his wife. He also cannot remember the names of his family members and familiar places. He is diagnosed with Alzheimer's disease. Which of the following nuclei is most likely degenerated? A. Anterior horn cell B. Basal ganglia C. Caudate nucleus D. Nucleus basalis of Meynert E. Thalamus

D. Nucleus basalis of Meynert

A 52-year-old man comes to the physician because of a 3-week history of weakness affecting his thighs and the face. He is diagnosed with an autoimmune condition that targets voltage-gated calcium channels found at the axonal terminals of lower motor neurons. Decreases in which of the following release-related events is most likely a feature of the disease? A. Probability of an action potential B. Magnitude of an action potential C. Duration of an action potential D. Release of ACh in response to an action potential

D. Release of ACh in response to an action potential

A 46-year-old man experiences occlusion of the left anterior inferior cerebellar artery, leading to deafness of the left ear with dizziness and nausea. Which of the following patterns of eye movements will be seen on attempted primary gaze. A. Tonic leftward deviation B. Tonic rightward deviation C. Slow rightward deviation followed by fast leftward resetting D. Slow leftward deviation followed by fast rightward resetting E. Fast rightward deviation of the eyes followed by slow leftward resetting F. Fast leftward deviation of the eye followed by slow rightward resetting

D. Slow leftward deviation followed by fast rightward resetting -left vestibular apparatus not fine and wants to push eyeballs to the left so we get a slow leftward deviation, but left frontal lobe doesn't like it so it snaps the eyeballs back to the right with a Saccadic movement

Which axon has the lowest conduction velocity? A. Large diameter, myelinated B. Medium diameter, myelinated C. Small diameter, myelinated D. Small diameter, unmyelinated

D. Small diameter, unmyelinated

A 25-year-old man is brought to the emergency department because of several pieces of broken glass lodged in his orbit following a bar fight. A CT scan of the head shows one of the pieces of glass has severed one of the nerves located in the orbit. After successful removal of the glass, physical examination shows decreased sensation to the forehead, scalp and upper eyelid. Which of the following nerves is most likely damaged? A. Oculomotor B. Great auricular C. Lacrimal D. Supraorbital E. Nasociliary

D. Supraorbital

A 60-year-old man comes to the emergency department because of a 17-year history of severe paroxysmal, electric-shock-like pain (visual analog scale = 10), located on the right side of the face, in the area corresponding to the mandibular branch of the trigeminal nerve, diagnosed as Idiopathic Trigeminal Neuralgia, according to the International Association for the Study of Pain criteria. Pharmacological treatment had been only minimally effective over the years. He could not eat, talk, shave, touch his face, cut his hair, and had not brushed his teeth over the past 6 years due to pain triggering. Which of the following structures is the most likely target for surgical intervention? A. Dorsal column B. Medial lemniscus C. Anterolateral system D. Trigeminal ganglion

D. Trigeminal ganglion

Localize lower motor neurons of the ventral horn operating extensor muscles of the wrist. A. Dorsomedial B. Dorsolateral C. Ventromedial D. Ventrolateral

D. Ventrolateral

A group of researchers is studying the effects of modified expression of a gene on development of the alar plate in the embryo. Following such modification, which of the following nuclei would most likely be most severely affected? A. Oculomotor B. Abducens C. Hypoglossal D. Vestibulocochlear

D. Vestibulocochlear

A 53-year-old woman is brought to the emergency department because of trouble speaking. Imaging shows an infarction affecting her left frontal lobe of the brain. Which additional clinical manifestation is likely? A. Tingling left hand B. Tingling right hand C. Weakness left hand D. Weakness right hand E. Tremor left hand F. Tremor right hand

D. Weakness right hand

Why do blood vessels appear black in DSAs?

Digital subtraction angiography- you take one x-ray without contrast and one with contrast and subtract the two

A bullet passes posteriorly in zonal plane, starting between the eyes of a crime victim. What is the most likely state of the pupils of the victim?

Dilated, so damage to the midbrain and damage to CN 3. You can see the cerebral peduncles and midbrain, and with the bullet it will hit the oculomotor, which does parasympathetics and prevent constriction of the pupils

Where is the origin of the excitatory graded potential?

Distal Dendrites

A healthy 24 year old man got trapped and had to fast for 6 days. He had sufficient fresh water so although hungypry, he was otherwise healthy when he was found. Prior to referring, which of the following fuel molecules supplied the majority of the ATP in the brain of this patient? A. Fatty acids from adipose stores B. Glutamine, glutamate, a-KG C. BCAA D. Glucose E. Ketone bodies

E. Ketone bodies

A 74-year-old man comes to the physician because of numbness affecting the left side of his face. Examination confirms insensitivity to somatosensory stimuli on the affected facial region. Imaging shows a small infarction affecting diencephalic cell bodies. Which of the following structures is injured? A. Central nerve B. Peripheral nerve C. Central tract D. Peripheral tract E. Central nucleus F. Peripheral nucleus

E. Central nucleus

A 42 year old man comes to the physician because of worsening 1 month history of inability to see things directly in from to from. He has a history of blunt trauma to his right eye 2 months ago. His vital signs are all WNL. PE shows loss of accommodation reflex in the affected eye and normal pupillary light reflex. Which of the following structures may have been damaged to result in the changes observed in this patient? A. Lens B. Retina C. Cornea D. Iris E. Ciliary muscle F. Sclera G. Choroid

E. Ciliary muscle

Which of the following is a feature of a lower motor neuron disease? A. Spasticity B. Hyperreflexia C. Babinski sign D. Clonus E. Fibrillations

E. Fibrillations - Need EMG because we can't detect with with a naked eye

A 47-year-old man is brought to the physician because of a 6-hour history of unsteadiness on his feet. Physical examination shows erratic eye movements. During an assessment of gait, the man sways and nearly loses his balance. Imaging studies show a cerebellar lesion. Which of the following cerebellar sites is most likely affected by the lesion? A. Anterior lobe B. Posterior lobe C. Interposed nuclei D. Dentate nucleus E. Flocculonodular lobe

E. Flocculonodular lobe

A 67-year-old man complains of a nagging earache for about one week unrelated to swallowing his food. Physical examination shows a normal outer ear canal and tympanic membrane with normal hearing tests. The possible cause of the earache could most likely be due to which of the following? A. An otitis media B. An impinged nerve between atlas and occipital bone C. An impinged nerve between the sixth and seventh vertebral body D. A cancerous growth on the hard palate E. A dental implant in the lower jaw one week ago

E. A dental implant in the lower jaw one week ago (a slight difference with proper occlusion causes signals in the TMJ and produces earache) A. An otitis media (not with a normal TM, hearing test and unlikely at that age) B. An impinged nerve between atlas and occipital bone (that would be C1, no contribution to ear) C. An impinged nerve between the sixth and seventh vertebral body (that would be C7 and does not contribute) D. A cancerous growth on the hard palate (that is CNVii and does not contribute to sensation of ear)

Neurodevelopment is associated with which of the following stages of human life? A. Embryonic B. Fetal C. Just before birth D. Just after birth E. All of the above

E. All of the above

When the chorda tympani in the middle ear is cut, it could lead to which of the following conditions? A. Pain (otalgia) B. Atrophy of parotid gland C. Loss of taste post. aspect tongue D. Dry mouth E. Altered taste sensation

E. Altered taste sensation - goes to ant. 2/3 of tongue

Injury to alpha motor neurons will likely result in which of the following signs? A. Babinski B. Clonus C. Hypertonia D. Spasticity E. Atrophy

E. Atrophy

A 72-year-old man is brought to the physician because of shaking in his right hand for 4 months. He has had decreased sense of smell for 8 months and his wife complains of his recent restlessness during sleep. Physical examination shows resting tremor of the right hand and slow shuffling gait. He has reduced facial expressions. In which of the following structures is the release of dopamine most likely reduced? A. Left cerebral cortex B. Right cerebral cortex C. Left thalamus D. Right thalamus E. Left striatum F. Right striatum

E. Left striatum

A 73-year-old man comes to the physician because of a 3-month history of problems hearing. Neurological examination shows: Weber's test lateralizes to the right, Rinne's test AC>BC on the right, AC > BC on the left but times reduced compared to the right. What is the most likely diagnosis? A. Left otosclerosis B. Right otosclerosis C. Left otitis media D. Right otitis media E. Left vestibular schwannoma F. Right vestibular schwannoma

E. Left vestibular schwannoma

A 79-year-old woman is brought to the emergency department because of an unsteady gait with clumsiness and incoordination of the left upper and lower limbs. The arrow indicates an infarction overlapping with a large structure within the posterior cranial fossa. Embryologically speaking, which part of the brain gives rise to the injured structure? A. Telencephalon B. Diencephalon C. Mesencephalon D. Myelencephalon E. Metencephalon

E. Metencephalon

A 52-year-old man comes to the physician because of a 1-year history of motor and sensory problems affecting the right hand. He is given a diagnosis of positive transection of the radial nerve. Physical examination shows that 50% of original strength has returned to the affected muscles. Which of the following organelles most likely mediates the advance of the growth cone during the repair process? A. Lysosomes B. Synaptic vesicles C. Microtubules D. Neurofilaments E. Microfilaments

E. Microfilaments

42-year-old man comes to the physician because of a 3-week history of headache and double-vision. Imaging studies show an aneurysm that is compressing the root of cranial nerves III. Which of the following structures is most likely affected by the aneurysm? A. Cerebrum B. Medulla C. Pons D. Diencephalon E. Midbrain

E. Midbrain

A 21 year old woman comes to the physician because of a 4 day history of developing increased weakness in her legs, she describes tingling of the feet and PE shows recused of two point discrimination in her legs. Her medical history if positive for pulmonary infection 3 weeks ago. Which of the following techniques can best be used to identify the pathological population of cells in this patient? A. Tensilon test B. EMG C. MRI D. PET E. NCV

E. NCV -can differentiate loss of fiber vs. loss of myelination. EMG purely within axon and better for NMJ

A 25 year old woman comes to her physician because of a 4 week history of facial weakness and blurred vision. The weakness and vision issues are worse towards the end of the day and resolved with rest. Neurological exam shows normal deep tendon reflexes and the absence of sensory deficits. Which of the following diagnoses is most likely? A. Soma B. Axon C. Schwann cells D. Muscle fibers E. Neuromuscular junction

E. Neuromuscular junction Applies to Myasthenia gravis d/t improvement with rest and worse towards the end of the day

***A 65-year-old man comes to the physician because of a 2-week history of urinary dribbling. He is recovering from injury to the sacral spinal segments. Imaging studies show damage of the anterior horn cell of the sacral spinal cord leading to denervation of the external urethral sphincter. Stimulation of which of the following types of receptors on his external urethral sphincter would most likely decrease the dribbling? A. Alpha B. Beta C. Dopamine D. Muscarinic E. Nicotinic

E. Nicotinic Micturition reflex supplied by parasympa. ANS internally, but externally it's skeletal muscle supplied by soma to motor nerve. Muscarinic are predominantly in involuntary organs.

A 57-year-old man presents to his physician concerned that he may have suffered a stroke. He explains that for the past two weeks he has had trouble with his vision and difficulty recognising faces. Following referral, neurological examination confirms prosopagnosia and shows left superior quadrantanopia. Word recognition is intact. What is the most likely location of the causative lesion? A. Right lingual gyrus B. Left lingual gyrus C. Right occipitotemporal gyri D. Left occipitotemporal gyri E. Right lingual and occipitotemporal gyri F. Left lingual and occipitotemporal gyri

E. Right lingual and occipitotemporal gyri

A 62-year-old man is brought to the emergency department because of a 2-hour history of weakness of his left arm and leg. Imaging studies show a vascular lesion affecting the crus cerebri. Which of the following arteries is most likely implicated in this lesion? A. Right anterior cerebral B. Left anterior cerebral C. Right middle cerebral D. Left middle cerebral E. Right posterior cerebral F. Left posterior cerebral

E. Right posterior cerebral

A 77-year-old man is brought to the physician by his spouse because of a 4-month history of memory problems. Examination shows incontinence and an awkward gait. He seems unconcerned by the problems. Imaging studies show hydrocephalus, suggesting dysfunction of the arachnoid granulations. With which of the following structures is the dysfunctional apparatus most likely linked? A. Choroid plexus B. Ependymal layer C. Cerebral aqueduct D. Central canal E. Superior sagittal sinus F. Interventricular foramina

E. Superior sagittal sinus A is what forms CSF B associated with choroid plexus C comes after 3rd ventricle and before 4th ventricle

A healthy 24 year old man participating in a research study which measures oxygen uptake and usage by neural tissues, which of the following cofactors if essential to support the biochemical pathway that is responsible for the majority of the consumption of oxygen in the brain of this individual? A. Pyridoxal phosphate B. Colbalamin C. Ascorbic acid D. BH4 E. TPP F. THF

E. TPP - related to a-KG, PDH, transketolase in PPP, BCAA-DH (deficiency leads to Maple Syrup disease) Colbalamin (B12) needed in homocysteine to methionine with folate (folate trap)

A 23-year-old woman with nasal problems has a recent CT that shows complete obliteration of the posterior ethmoid air cells by nasal polyps without bony erosion. This location has a potential severe complication when she goes for operation to have these polyps removed. Which of the following structures that is close to those air cells must be avoided? A. sphenopalatine artery B. nasopalatine nerve C. anterior ethmoidal artery and nerve D. posterior ethmoidal artery and nerve E. optic nerve

E. optic nerve

Discuss the causes of epidural versus subdural hemorrhages and how to differentiate these hemorrhagic event in the neuroimages. Why don't epidural bleeds spread?

Epidural hemorrhages are traumatic and instant whereas subdural are due to trauma that happened weeks ago. Epidural is in the younger age group looking like a football and subdural looks like a crescent moon. Epidural bleeds don't spread because of the sutures

Guillain-Barre etiology and clinical findings associated with peripheral neuropathy (both positive and negative symptoms)

Etiology: autoimmunity implicated post infectious (bacterial/viral) immune mediated disease that causes demyelination of peripheral axons Clinical findings: meningitis vaccine, acute inflammatory polyradiculoneuropathy d/t respiratory or GI infection

Leprosy etiology and clinical findings associated with peripheral neuropathy (both positive and negative symptoms)

Etiology: infecting of skin and peripheral nerves by mycobacterium leprae (long incubation or slow multiplication) with bacterial entry into cutaneous lesion followed with nasal secretions contact (PROFOUND SENSORY LOSS OF PAIN AND TEMP, AND TRAUMA TO EXTREMITIES) Clinical findings: Painless skin patch with wasting and muscle weakness

Lead poisoning etiology and clinical findings associated with peripheral neuropathy (both positive and negative symptoms)

Etiology: motor neuropathy (ADULTS) and encephalopathy (CHILDREN UNDR 6) Clinical findings: Bilateral focal weakness and wasting of extensor muscles of fingers, wrist and arms

Diabetic neuropathy etiology and clinical findings associated with peripheral neuropathy (both positive and negative symptoms)

Etiology: numbness/tingling/pain starts at distal extremities and moving proximally (STOCKING AND GLOVE DISTRIBUTION), diabetic foot with necrotizing abscess and wound infection, Charcot Neuropathic Osteoarthropathy (CN) Clinical findings:

Carpal tunnel etiology and clinical findings associated with peripheral neuropathy (both positive and negative symptoms)

Etiology: occurs spontaneously in middle aged women, pregnancy or hypothyroidism debt reduction of carpal tunnel size from swelling of tendon sheath as a result of overuse/edematous soft tissue Clinical findings: pain/tingling sensation felt in the digital cutaneous distribution of median nerve, difficulty in handling small objects, sometimes with wasting of a doctor brevis and opponents pollicis associated with atrophy of thenar eminence

Explain the relations between visual fields and retinal fields

Everything up/down is flipped; everything left/right is foipped

A 27-year-old man comes to the physician because of a 1-month history of problems hearing. Weber's test does not lateralize. Rinne's test shows R: BC > AC, L: BC > AC. What is the most likely diagnosis? A. Left otosclerosis B. Right otosclerosis C. Left vestibular schwannoma D. Right vestibular schwannoma E. Bilateral Meniere's disease F. Bilateral cerumen impaction

F. Bilateral cerumen impaction

A 68-year-old woman is brought to the emergency department because of a 3-hour history of weakness in the left arm and leg. Physical examination shows unilateral left hemiparesis, left hyperreflexia and a left extensor plantar response. Which of the following receptors is most likely linked to the afferent limb of the plantar reflex? A. Pacinian corpuscle B. Merkel's disk C. Meissner's corpuscle D. Golgi tendon organ E. Muscle spindle F. Free nerve ending

F. Free nerve ending

An investigator assesses the length constant of somatosensory axons. She confirms that the length constants of the axons vary depending on the somatosensory modality. Specifically, fibers conducting information related to pain have shorter length constants than do fibers encoding muscular stretch. Which of the following patterns of electrical resistance is most likely expressed by pain fibers bearing the shortest length constants? A. High intracellular B. Low intracellular C. High transmembrane D. Low transmembrane E. High intracellular and high transmembrane F. High intracellular and low transmembrane G. Low intracellular and high transmembrane H. Low intracellular and low transmembrane

F. High intracellular and low transmembrane

A 42-year-old man is brought to the emergency department because of traumatic injuries. Reflex testing shows the pattern indicated below. Cranial nerve function is intact. At which of the following sites is the lesion most likely located? A. Right internal capsule B. Left internal capsule C. Right pons D. Left pons E. Right cervical cord F. Left cervical cord G. Right lumbar cord H. Left lumbar cord Right. Left Biceps. 1+. 0 Triceps. 2+. 0 Patellar 2+/3+ Ankle 2+/4+

F. Left cervical cord - look at the 0s because that's where most of the damage is

A 77-year-old man is brought to the emergency department because of severe headache and nausea for 3 days. An MRI shows a mass on the lateral aspect of his left frontal lobe. Which of the following signs is most likely in this patient? A. Left biceps reflex 4+ B. Left triceps reflex 0 C. Left ankle jerk reflex 1+ D. Right extensor plantar response E. Right knee jerk reflex 0 F. Right brachioradialis reflex 4+

F. Right brachioradialis reflex 4+

A 64-year-old woman is brought to the emergency department because of a 1-day history of abnormal movements of her left upper limb. She is unable to control the limb when the movement begins. Physical examination shows an overweight woman with periodic uncontrollable flailing and rotatory movements of her left upper extremity. Which of the following structures is most likely damaged? A. Left substantia nigra B. Left subthalamic nucleus C. Left precentral gyrus D. Right precentral gyrus E. Right substantia nigra F. Right subthalamic nucleus

F. Right subthalamic nucleus -flailing and rotatory movements is a sign of Hemiballismus

***A 61-year-old woman comes to the physician because of a 4-day history of sudden-onset difficulty hearing. She compensates for the difficulty by turning up the volume on her tablet. Comprehension is difficult during conversation. Imaging studies show bilateral abnormalities overlapping with the dorsal brainstem. Which of the following structures is most likely damaged? A. Thalamus B. Splenium C. Tegmentum D. Cerebellum E. Lingula F. Tectum

F. Tectum

What are the advantages of having a fast active transport mechanism in neurons?

Faster transport of processes; We need to recycle post synaptic and presynaptic terminals which do anterograde transport, and we get neuro terminals at the soma, which is good for exocytosis

Why is the area with the highest NaV density the most sensitive area for AP initiation?

For AP once it's initiates we depolarize all the Na channels so signal is moved fro one channel to another. If the distance is too wide, then there's a loss of signal. If there's high NaV density and the channels are close to each other, the signal gets to its destination faster and less net loss

List the three major parts of the brainstem and discuss their relative location from rostrum to caudal. What are the anatomical positions of the brain? Define the function of the brainstem.

From Rostral to Caudal: Midbrain, pons, medulla oblongata, spinal cord Due to embryological folding: Rostral - top after diencephalon Caudal - bottom Dorsal - top of head Ventral - bottom Brainstem controls respiration, blood pressure, swallowing, consciousness and contains most of your cranial nerves except I (olfactory) and II (optic)

A 21-year-old man is brought to the emergency department by ambulance because of a 1-hour history of a penetrating head-wound. Imaging studies show damage to the right parietal lobe. Which of the following visual defects is most likely expected? A. Bitemporal hemianopia B. Binasal hemianopia C. Left homonymous hemianopia D. Right homonymous hemianopia E. Left superior quadrantanopia F. Right superior quadrantanopia G. Left inferior quadrantanopia H. Right inferior quadrantanopia

G. Left inferior quadrantanopia

A 62-yr-old man develops slowed movements of his left hand and a resting tremor, prompting preliminary a diagnosis of Parkinson's disease. The clinical presentation reflects damage in the midbrain, leading to dysregulation of which of the following cerebral lobes. A. Right occipital B. Left occipital C. Right temporal D. Left temporal E. Right parietal F. Left parietal G. Right frontal H. Left frontal I. Right insular J. Left insular

G. Right frontal

A 23-year-old woman comes to the physician because of a 3-day history of a painful rash. Which of the following dermatomes is most likely affected? A. T3 B. T4 C. T5 D. T6 E. T7 F. T8 G. T9 H. T10 I. T11 J. T12

G. T9

A 71-year-old man is brought to the emergency department because of a fall. Imaging shows a cortical infarction reflecting occlusion of the left middle cerebral artery. Which of the following clinical manifestations is/are most likely? A. Weak arm B. Weak leg C. Numb arm D. Numb leg E. Weak arm and leg F. Numb arm and leg G. Weak and numb arm H. Weak and numb leg I. Weak and numb arm and leg

G. Weak and numb arm Suggestion: MCA dominates lateral cerebral surface so motor and sensory could be affected. ACA affects anteriomedial surface, and PCA affects posterior-medial surface

A patient experiences right inferior macular quadrantanopia secondary to a cortical lesion. Localize the injury. A. Right anterior lingula B. Left anterior lingula C. Right inferior occipital pole D. Left inferior occipital pole E. Right anterior cuneus F. Left anterior cuneus G. Right superior occipital pole H. Left superior occipital pole

H. Left superior occipital pole

Outline the differences between Hemiballismus and Huntington's disease

Hemiballismus - Not genetic; infarction; on one side of the body Huntington - Genetic (AD); whole body abrupt and jerky movement (dance like movements)

What is the sensory innervation of the Golgi tendon organ?

Ib fiber

What is the difference between hypokinesis and hyperkinesis?

Hypokinesis: diminished or abnormally slow movement Hyperkinesis: excessive movement, abnormal uncontrolled muscular activity

A 19-year-old man comes to the physician 2 hours after suffering a traumatic knee injury. A local anesthetic is applied via lumbar puncture in preparation for arthroscopic surgery. In addition to anesthesia, the drug induces bilateral paralysis of the lower limbs. Which of the following neuron/channel pairs is mostly likely relevant to the paralytic effects of the drug? Neuron Channel A. P Calcium B. P Sodium C. P Chloride D. P Magnesium E. P. Potassium F. Q Calcium G. Q Magnesium H. Q Chloride I. Q Sodium J. Q Potassium

I. Q Sodium

Why are cerebellar signs ipsilateral?

If they cross over, they cross over twice; they're always on the same side of the lesion

Discuss what would happen if you would pharmacologically selectively block A. A-delta fibers B. C fibers How would the persons pain perception change, compared to normal, under those conditions?

If you block them, you wouldn't have perception of pain. A delta fibers are smaller and myelinated so they run faster (rapid, sharp, highly localized onset of pain), and your C fibers (longer, slower and duller lasting pain) are larger and unmyelinated and run slower. The one used for pain 1st pain: A-delta fibers 2nd pain: C fibers

Why is the action potential not graded but an all or none event?

If you have enough stimulus to cause AP, you want it to react and you want a max depolarization. It ensures the AP is propogated to its max, so you might need multiple small triggers for an AP. In all, for the receptors to start firing you need triggers for an AP and max depolarization and you don't want to delay the trigger because something else can trigger it

Explain the role of Ach-esterase in Cholinergic neutotransmission

In the synaptic cleft, provides fast degradation of Ach so there's no desensitization and there's tetany. It basically eats ACh to reduce the lifespan of the NT so it doesn't activate the postsynaptic cleft (why people have MG)

Contrast the causes of perigentricular versus subarachnoid hemorrhages and how to differentiate these hemorrhagic events in neuroimages.

Intracerebral collects around parenchyma, but blood actually collects in periventricular, but blood comes from the same source for both. Periventricular has blood accumulate in the intraventricular system Subarachnoid is a cerebral artery rupture between the arachnoid and the pia mater and blood goes into the sulci. Can be caused by trauma or spontaneous cause (ex. aneurysm)

What is the function of the muscle spindle in the inverse myotatic reflex?

Inverse myotatic reflex - when muscle tension gets high, the Golgi tendon organ reflex limits the contraction of the muscle (gives out INHIB INTERNEURONS), so the Golgi tendon organ reflex REGULATES muscle tension. So the function of the muscle spindle is to change in length or create tension

Differentiate ionotropic and metabotropic receptors

Ionotropic are Nicotinic and control ion flux between a membrane through a pore (faster). Metabotropic receptors are muscarinic and coupled by GPCR without the use of a pore (slower and longer)

What are the two main routes for glutamate from the synaptic cleft in a tripartite synapse? What are the benefits of astrocytic uptake of glutamate?

It can go back into the vesicular transporter or go through the glutamate transporter into the astrocyte and made into glutamine. The benefit is that with making glutamine in the astrocyte, you're able to recycle it back into the presynaptic terminal and use it to make more glutamate

Reason why should the poly neural innervation be converted to mono neural innervation? Discuss the postnatal experience based editing of the developing nervous system.

It optimizes the initiation of the synapses. Synaptic proline causes efficiency of transmission of synapses. Post natal is when the neuron that's more dominant is the neuron that persists, and segments into different regions. So the left eye gets innervation from one side and the same to the right, but they won't cross over. If a child has loss of vision in one eye, the other eye would have both signals go to the eye that's working and it would be underdeveloped. If one eye hasn't been exposed to the environment, only the eye exposed would be able to perceive image and the other eye would be underdeveloped. If both eyes were exposed to environment in critical period, then both es would be able to perceive images and information (SYNAPTIC PROLINE).

In which direction would a positively charged transmembrane protein be pushed when a excitable cell depolarizes?

It pushes the transmembrane protein out; the positively charged AA move outwards on depolarization

Why would lesions to the lateral geniculatr nuclei cause blindness?

It's where vision comes from to enter into optic radiation to the brain, so optic tracts enter that so you don't have any connection to optic radiation or occipital lobe

Bilateral lesions associated with which of the artery would most likely cause deficits in the perception of depth and motion?

MCA around parieto-occipital area

Which CNS cell type is part of the immune system?

Microglia

What are the potential consequences of abnormalities of nicrotubules?

Microtubules are used for the shape of the axon and transportation through polymerization. If abnormal, you would have difficulty with axonal transport (ex. Alzheimer's disease)

Why would loss of myelination make it more likely that AP conduct fails entirely?

Myelin insulates the axon. Without it, the axon potential would have to depolarize the whole membrane instead of doing it in sections, which would cause the signal to take longer to get to its destination and therefore loss of signal

**Discuss the important neural induction proteins? What are the consequences of dysfunctional expression of genes regulating neural induction? Discuss the basis of formation of the PNS

NS from the ectoderm. Neural crest cells develop PNS. Neural induction proteins Noggin and Chordin, which helps Rostral and caudal parts of the neural tube fold in itself while neural crest cells form. As the two completely close, Rostral part becomes the 3 vesicles (prosencephalon, mesencephalon, and rhombocephalon) Consequences: holoprosencephaly, spina bifida, schizencephaly

What are the potential consequences of abnormalities of neurofilaments?

Neurofilaments determine atonal diameter. If abnormal, axon wouldn't be thick enough and the nerve can't conduct as well (slow generation)

A 29 year old woman was given isotritinoin for severe Calcutta NT acne. During the first 3 months of treatment, she was taking an oral contraceptive agent that she had started at the same time as the isotrtinoin. After deciding she wanted to become pregnant, the patient stopped taking the contraceptive agent but continued to take isotretinoin for another 2 months,. Should she continue? Why?

No because it's mainly retinoids acid and that's a teratogen because it messes with the retinoids balance in the mom, so more accumulates and too much is given to the fetus, which causes facial deformities, especially in the eyes.

Distinguish between nuclei and tracts.

Nuclei are clusters of functionally related central neuronal cell bodies (synthesize NTs) and tracts are bundled axons for communication between nuclei (mediate release of NTs)

What cells are forming the myelin sheath in the CNS?

Oligodendrocytes

What aspect of hearing would be affected where there is degeneration of the ossicles?

Ossicles serve to transmit sounds from the air to the fluid-filled labyrinth (cochlea), so you wouldn't be able to amplify sound in the middle ear, leading to conductive hearing loss

Relate Parkinsonism to the direct and indirect pathways.

Parkinson's disease = insufficient dopamine = don't have enough to bind to D1 or D2 receptors, so you don't inhibit the neostriatum or stimulate it D1 —> cortex excites neostriatum = more GABA made = GPi inhibited and disinhibits the thalamus = activates thalamus and excites cortex for more movement D2 —> cortex excites neostriatum = more GABA made = GPe is inhibited so can't make enough GABA = subthalamus active and free to stimulate GPm which makes tons of GABA = inhibit thalamus and less movement

Due to intolerable pain stemming from a left-sided pelvic tumor, a quadrant of the spinal cord is surgically sectioned. Which quadrant is targeted by the neurosurgeon?

Right ventral/anterolateral quadrant from the anteriolateral system (when it enters the spinal cord it crosses over from the medulla)

Despite the tight junctions in the epithelial lining blocking paracellular transport, which type of substances would you expect to still be able to passively diffuses across the BBB? a. Ions b. AA c. Lipid-soluble molecules d. Hydrophilic molecules e. Water

c. Lipid-soluble molecules (heroin, nicotine, ethanol, diazepam; morphine can't cross the BBB)

Which cells are forming the myelin sheath of the PNS?

Schwann cells; promotes regeneration of axons for injury repair by making a guiding tube

Why would loss of myelination slow AP conduction velocity?

Signal can't jump to Nodes of Ranvier which means loss of conductivity and loss of the signal

What are the similarities and differences between CSF and blood plasma?

Similarities: Isotonic osmolarity and same Na+ levels Differences: Low protein and glucose, low potassium and calcium, high magnesium and chloride ion, and low bicarbonate and pH

What is the function of glutamate and GABA in the basal ganglia pathway?

Stimulates with glutamate and inhibits with GABA

Review the differential effect of dopamine on the basal ganglia

Stimulates/modulates movement -Stimulates the direct pathway and inhibits the indirect pathway

What visual fields defect would result form damage to the right Meyer's loop?

Superior Left homonymous quadrantanopia (PIE IN THE SKY)

What two sets of paired arteries arise from. The Rostral end of the basilar artery?

Superior cerebral artery and the posterior cerebral artery. Basilar artery forms from the ventral surface of the pons and comes from the vertebral artery

What are the major roles of the cerebellum?

Synergy of movement, posture, balance, coordination, motor functions and cognition (language, prediction); any mental concepts you think about are processed by cerebellum

Compare and contrast the mechanism and resulting motor responses to strychnine poisoning and tetany toxin

Tetany toxin suppresses release of glycine PRESYNAPTICALLY from bulbar and spinal interneurons, so lower motor neurons are DISINHIBITED =THERES INCREASED MUSCLE CONTRACTION (muscle becomes fatigued and tired = weakness) Strychnine positioning - alkaloid blocks the glycine receptor POSTSYNAPTICALLY on the lower motor neuron, reducing chloride influx, so lower motor neurons are DISINHIBITED =THERES INCREASED MUSCLE CONTRACTION (muscle becomes fatigued and tired = weakness)

What part of the diencephalon contains nuclei that act as sensory relays? What are the other parts of the diencephalon?

Thalamus: touch, taste, temperature, pressure, pain Hypothalamus: homeostasis, growth and reproduction Third ventricle: fluid filled space of CSF CN II Subthalamus: motor and sensory function Epithalamus: endocrine function

What is the effect of the chief output nucleus of the basal ganglia?

The GPi/GPm is the inhibitory input that inhibits the thalamus, dampening the thalamus excitatory stimulation of the cortical neurons and diminishes movement

What is the relationship between the optic nerve and the blind spot?

The blind spot is where the optic nerve goes through. Optic nerve is enclosed and surrounded by dura

Why does strength increase eith sustained or repeated effort in patients with Lambert Eaton syndrome?

There's impaired Ca2+, so Ca2+ accumulates which causes more contractions and promote the release of Ach using 4-Aminopyrimidine so you have inc. NT release and inc. strength

Why does a smaller diameter axon offer more resistance to Intro axonal ion flux?

Think of a garden hose: smaller distance means more resistance

Explain the role of the D2 receptor antagonist in the treatment of Huntington's disease.

This is a hyperkinetic disorder, so you want to diminish movement in the thalamus with D2 receptor (decreasing the effect of the indirect pathway) D2 receptor antagonist blocks the indirect pathway, so neostriatum active & makes a lot of GABA, inhibiting GPl and stimulating the subthalamus, which then activating GPm which makes a lot of its own GABA and inhibit the thalamus

Where do we find upper and lower motor neurons in the spinal cord?

UMN: descending in lateral (90% of corticospinal tract) anterior funiculus LMN: ventral horn

The ventral column of white matter in the left side of the spinal cord is damaged. Which deficit will arise?

Ventral white matter carries pain and temperature sensation in the anterior-lateral pathway (spinothalamic tract). So pain and temp are lost from the contralateral (right) side

Describe the general cortical distribution of blood arising from the vertebral artery?

Vertebral artery supplies the Posterior inferior cerebellar arteries (PICA), which supplies the arteries to the vertebral comuln

Why do we see better in the center of the visual field compared to the periphery?

Want to focus it on the foveal centralis as it has the most cones (photoreceptors)

Which parts of acquired immunity are present in the CNS under normal conditions?

We don't have antibodies, T cells or B cells, so there's no part of acquired immunity (unless BBB is damaged) but microglia can come up as antigen presenting cells

There is a difference between the action of an extra ocular muscle and testing the function of that muscle. For instance: superior oblique (SO) muscles action and down. However, testing in and down. Why is that so?

We're trying our best to isolate the muscles and nerves under investigation. So by asking them to look in and down, we're trying to find the superior oblique muscle

During quantal analysis, why is a low calcium extracellular environment conductive to the release of transmitter very small numbers of vesicles?

When there's low extracellular calcium levels, it ensures there's a specific number of calcium needed for exocytosis. The release of Ca2+ controls the release of NTs. The extracellular environment keeps Ca2+ as a regulator so there's regulated amount of NTs that are being exocytosis If you reduced extracellular calcium by replacing it with magnesium, that means the only calcium available to regulate NT release is what is left in the environment. If you had more calcium, then your end plate potential is higher because you have more NT release. SO CALCIUM EGULATES OYUR END PLATE POTENTIAL.

A. How would lesions of SENSORY fibers of a peripheral nerve affect the myotatic reflex? B. How would lesions of MOTOR fibers of a peripheral nerve affect the myotatic reflex?

When you do patellar reflex, that's the stimulus stretching quads, so intrafusal fibers (Ia and II) generate AP and illicit myotatic reflex, and takes info to level of spinal cord where, on same side, it has excitatory impulses stimulating alpha motor neurons for contraction while going to the other side for reciprocal inhibition of the antagonist muscle using Ia fiber (hamstrings relax) A. Afferent sensory neurons ascends to cortex, communicates with UMN and controls the reflex by stimulating inhib. Interneurons it decrease that reflex. If you lose those sensory fibers and UMN reflexes, muscle contraction is NT regulates so it can't be toned down and it leads to HYPERREFLEXIA B. If you lose motor aspect fo myotatic reflex, you have HYPOREFLEXIA, meaning you won't be contracting the muscle (HYPOTONIA)

What are the pathways for formation of glutamine or GABA from glutamate? Which metabolic cyclic pathway contains a-KG as an intermediate? True/false: as long as a cell has adequate supply of glucose and a nitrogen source, glutamate may be formed as needed

a. Glutamine - need glutamate synthetase and ATP and a glutamine transporter GABA - need glutamate decarboxylase and PLP b. Can make glutamine for glutaminase, or a-KG through transamination or reductive amination c. True: pyruvate —> TCA cycle to make a-KG and that can be transaminated into glutamate

Imagine a cell has a Vm = -65 mV at rest and chloride concentration gradients that sets the Nernst potential for Cl- at E(Cl-) = -65 mV. What happens to Vm if opening of GABA(A) receptors adds additional Cl- conductance? If a depolarizing Na+ conductance is added, will the cell depolarize more or less easily in the presence of the additional Cl- conductance?

a. The Nernst potential for Cl- is at equilibrium with the membrane potential so there won't be any flow or net change, but Cl- channels will be open b. It will make it less easy to depolarize because of the additional chloride conductance. In a soccer game, the team with the less players has less of a chance of winning. So as we increase chloride conductance, you make it harder for the cell to depolarize and GABA (and glycine too) becomes INHIBITORY since you have more Cl- entering the cell

With the failure of the active transporters, ____ accumulates in neurons, leading to increased volumes of __________ fluid. a. K+; intracellular b. K+; extracellular c. Na+; intracellular d. Na+; extracellular e. Cl-; intracellular f. Cl-; extracellular g. Ca++; intracellular h. Ca++; extracellular

c. Na+; intracellular


Ensembles d'études connexes

Geography 4.02: Where Is Africa?

View Set

Les kanjis dans la tête - kanji vers prononciation/signification

View Set

Translate the sentences with "ser" (am, is are) into English.

View Set

Academic Team Full Practice Set 3

View Set

BECOMING HUMAN: Theories of emotion

View Set

ATI_AQ-Leadership_and_Management

View Set

Ch1: Life Insurance: General Insurance

View Set